- Tue Jun 02, 2020 11:35 am
#75880
Hi younghoon27!
Yes, your intuition is correct that there isn't enough evidence to support answer choice (A).
To see why (B) is better, first take note of the wording of the question: "The academic researchers mentioned in lines 30–31 would be most likely to subscribe to which one of the following principles?" This tells us (1) that the question is about a specific subject's perspective, (2) it directs readers to a specific line reference, and (3) it asks what we can infer from the passage about this subject's viewpoint (i.e., since it's asking for an inference, it's asking what "must be true" about this viewpoint/perspective based on the passage.
So we're therefore dealing with a must be true question that directs readers to "academic researchers" mentioned in lines 30-31. When given a specific line reference, it's important to read before and after those lines as well, for the purposes of placing this reference in a broader context. Here, the full sentence is: "Academic researchers who oppose biotechnology patents fear that corporate patent holders will charge prohibitively high fees for the right to conduct basic research involving the use of patented materials" (lines 30-34).
In other words, in this part of the passage, academic researchers are discussed as disliking fees for conducting basic research using patented materials. This is what is captured in answer choice (B): "The inventor of a biological material should not be allowed to charge fees that would prevent its use in basic research." Since the passage tells us that these academic researchers dislike fees that are high enough to create barriers to basic research, we can infer from the passage that such researchers think inventors should not impose such fees that prevent uses for basic research.
By contrast, not only does this part of the passage not talk about the market and thus does not provide enough evidence to know (A) is true, it actually seems that the next paragraph provides reason for concluding that the mentioned academic researchers would not agree with it. Answer (A) states, "The competitive dynamics of the market should be allowed to determine the course of basic scientific research." Within a trajectory in which "freely sharing research materials has shifted to a market model" (lines 36-37), academic researchers would prefer the former (freely sharing materials, so they can engage in basic research) to the latter (a market model in which inventors can set fee prices that stifle basic research). Since the question asks about what we can infer that the academic researchers would agree with, we can eliminate (A), since the passage suggests they would not agree with it.